Question and Answers Forum

All Questions      Topic List

Mechanics Questions

Previous in All Question      Next in All Question      

Previous in Mechanics      Next in Mechanics      

Question Number 29016 by ajfour last updated on 03/Feb/18

Commented by ajfour last updated on 03/Feb/18

Find maximum value of m  if bigger sphere has to remain  in contact with wall.Assume  friction coefficient large enough  at all surfaces.  r, R constant ;  r < R .

$${Find}\:{maximum}\:{value}\:{of}\:\boldsymbol{{m}} \\ $$$${if}\:{bigger}\:{sphere}\:{has}\:{to}\:{remain} \\ $$$${in}\:{contact}\:{with}\:{wall}.{Assume} \\ $$$${friction}\:{coefficient}\:{large}\:{enough} \\ $$$${at}\:{all}\:{surfaces}.\:\:{r},\:{R}\:{constant}\:; \\ $$$${r}\:<\:{R}\:. \\ $$

Answered by ajfour last updated on 03/Feb/18

Commented by ajfour last updated on 03/Feb/18

sin θ =((R−r)/(R+r))  ,  f_g =f   and   f=f_w   considering  zero torque on both spheres  individually about their centres.      f_g =N_w     considering net   horizontal force on system  consisting of both spheres.  so       f_g =f_w =f=N_w        eq.(A)    (do keep in mind )  net horizontal force on system     f_w +N_g =(M+m)g       ⇒     f+N_g  = (M+m)g   ....(i)  ΣF_x =0   for smaller sphere   N_w +fcos θ=Nsin θ  ⇒    f(1+cos θ)=Nsin θ   ....(ii)  ΣF_y =0  , so     f_w +Ncos θ+fsin θ=mg  ⇒ f(1+sin θ)+Ncos θ =mg  ...(iii)  ΣF_x =0   for larger sphere  fcos θ+f_g = Nsin θ  ⇒     f(1+cos θ) =Nsin θ   [same as (i)]  ΣF_y =0 , so  fsin θ+Mg+Ncos θ = N_g   Torque about corner of ground  and wall gives:  MgR+mgr+N_w [R+(R+r)cos θ]                    = N_g R             .....(iv)  we have unknowns     f, m, N, N_g    and eq.s  (i) to (iv)  using (ii) in (iii):    N[((sin θ(1+sin θ))/(1+cos θ))+cos θ]=mg  ⇒  N=((mg(1+cos 𝛉))/(1+cos 𝛉+sin 𝛉))         f = ((mgsin 𝛉)/(1+cos 𝛉+sin 𝛉))   =N_w          N_g = (M+m)g−f  using these in (iv):  MgR+mgr+f[R+(R+r)cos θ]               =(M+m)gR−fR  ⇒ mgr+f[2R+(R+r)cos θ]−mgR       =Mg(R−r)   ((mgsin θ[2R+(R+r)cos θ])/(1+sin θcos θ))−mg(R−r)                                            =Mg(R−r)  m=((M(R−r))/([((sin θ[2R+(R+r)cos θ])/(1+sin θcos θ)) − (R−r)]))  .

$$\mathrm{sin}\:\theta\:=\frac{{R}−{r}}{{R}+{r}}\:\:, \\ $$$${f}_{{g}} ={f}\:\:\:{and}\:\:\:{f}={f}_{{w}} \:\:{considering} \\ $$$${zero}\:{torque}\:{on}\:{both}\:{spheres} \\ $$$${individually}\:{about}\:{their}\:{centres}. \\ $$$$\:\:\:\:{f}_{{g}} ={N}_{{w}} \:\:\:\:{considering}\:{net}\: \\ $$$${horizontal}\:{force}\:{on}\:{system} \\ $$$${consisting}\:{of}\:{both}\:{spheres}. \\ $$$${so}\:\:\:\:\:\:\:\boldsymbol{{f}}_{\boldsymbol{{g}}} =\boldsymbol{{f}}_{\boldsymbol{{w}}} =\boldsymbol{{f}}=\boldsymbol{{N}}_{\boldsymbol{{w}}} \:\:\:\:\:\:\:{eq}.\left({A}\right) \\ $$$$\:\:\left({do}\:{keep}\:{in}\:{mind}\:\right) \\ $$$${net}\:{horizontal}\:{force}\:{on}\:{system} \\ $$$$\:\:\:{f}_{{w}} +{N}_{{g}} =\left({M}+{m}\right){g}\:\:\:\:\: \\ $$$$\Rightarrow\:\:\:\:\:{f}+{N}_{{g}} \:=\:\left({M}+{m}\right){g}\:\:\:....\left({i}\right) \\ $$$$\Sigma{F}_{{x}} =\mathrm{0}\:\:\:{for}\:{smaller}\:{sphere} \\ $$$$\:{N}_{{w}} +{f}\mathrm{cos}\:\theta={N}\mathrm{sin}\:\theta \\ $$$$\Rightarrow\:\:\:\:{f}\left(\mathrm{1}+\mathrm{cos}\:\theta\right)={N}\mathrm{sin}\:\theta\:\:\:....\left({ii}\right) \\ $$$$\Sigma{F}_{{y}} =\mathrm{0}\:\:,\:{so} \\ $$$$\:\:\:{f}_{{w}} +{N}\mathrm{cos}\:\theta+{f}\mathrm{sin}\:\theta={mg} \\ $$$$\Rightarrow\:{f}\left(\mathrm{1}+\mathrm{sin}\:\theta\right)+{N}\mathrm{cos}\:\theta\:={mg}\:\:...\left({iii}\right) \\ $$$$\Sigma{F}_{{x}} =\mathrm{0}\:\:\:{for}\:{larger}\:{sphere} \\ $$$${f}\mathrm{cos}\:\theta+{f}_{{g}} =\:{N}\mathrm{sin}\:\theta \\ $$$$\Rightarrow\:\:\:\:\:{f}\left(\mathrm{1}+\mathrm{cos}\:\theta\right)\:={N}\mathrm{sin}\:\theta\:\:\:\left[{same}\:{as}\:\left({i}\right)\right] \\ $$$$\Sigma{F}_{{y}} =\mathrm{0}\:,\:{so} \\ $$$${f}\mathrm{sin}\:\theta+{Mg}+{N}\mathrm{cos}\:\theta\:=\:{N}_{{g}} \\ $$$${Torque}\:{about}\:{corner}\:{of}\:{ground} \\ $$$${and}\:{wall}\:{gives}: \\ $$$${MgR}+{mgr}+{N}_{{w}} \left[{R}+\left({R}+{r}\right)\mathrm{cos}\:\theta\right] \\ $$$$\:\:\:\:\:\:\:\:\:\:\:\:\:\:\:\:\:\:=\:{N}_{{g}} {R}\:\:\:\:\:\:\:\:\:\:\:\:\:.....\left({iv}\right) \\ $$$${we}\:{have}\:{unknowns} \\ $$$$\:\:\:{f},\:{m},\:{N},\:{N}_{{g}} \:\:\:{and}\:{eq}.{s}\:\:\left({i}\right)\:{to}\:\left({iv}\right) \\ $$$${using}\:\left({ii}\right)\:{in}\:\left({iii}\right): \\ $$$$\:\:{N}\left[\frac{\mathrm{sin}\:\theta\left(\mathrm{1}+\mathrm{sin}\:\theta\right)}{\mathrm{1}+\mathrm{cos}\:\theta}+\mathrm{cos}\:\theta\right]={mg} \\ $$$$\Rightarrow\:\:\boldsymbol{{N}}=\frac{\boldsymbol{{mg}}\left(\mathrm{1}+\mathrm{cos}\:\boldsymbol{\theta}\right)}{\mathrm{1}+\mathrm{cos}\:\boldsymbol{\theta}+\mathrm{sin}\:\boldsymbol{\theta}} \\ $$$$\:\:\:\:\:\:\:\boldsymbol{{f}}\:=\:\frac{\boldsymbol{{mg}}\mathrm{sin}\:\boldsymbol{\theta}}{\mathrm{1}+\mathrm{cos}\:\boldsymbol{\theta}+\mathrm{sin}\:\boldsymbol{\theta}}\:\:\:=\boldsymbol{{N}}_{\boldsymbol{{w}}} \\ $$$$\:\:\:\:\:\:\:\boldsymbol{{N}}_{\boldsymbol{{g}}} =\:\left(\boldsymbol{{M}}+\boldsymbol{{m}}\right)\boldsymbol{{g}}−\boldsymbol{{f}} \\ $$$${using}\:{these}\:{in}\:\left({iv}\right): \\ $$$${MgR}+{mgr}+{f}\left[{R}+\left({R}+{r}\right)\mathrm{cos}\:\theta\right] \\ $$$$\:\:\:\:\:\:\:\:\:\:\:\:\:=\left({M}+{m}\right){gR}−{fR} \\ $$$$\Rightarrow\:{mgr}+{f}\left[\mathrm{2}{R}+\left({R}+{r}\right)\mathrm{cos}\:\theta\right]−{mgR} \\ $$$$\:\:\:\:\:={Mg}\left({R}−{r}\right) \\ $$$$\:\frac{{mg}\mathrm{sin}\:\theta\left[\mathrm{2}{R}+\left({R}+{r}\right)\mathrm{cos}\:\theta\right]}{\mathrm{1}+\mathrm{sin}\:\theta\mathrm{cos}\:\theta}−{mg}\left({R}−{r}\right) \\ $$$$\:\:\:\:\:\:\:\:\:\:\:\:\:\:\:\:\:\:\:\:\:\:\:\:\:\:\:\:\:\:\:\:\:\:\:\:\:\:\:\:\:\:={Mg}\left({R}−{r}\right) \\ $$$${m}=\frac{{M}\left({R}−{r}\right)}{\left[\frac{\mathrm{sin}\:\theta\left[\mathrm{2}{R}+\left({R}+{r}\right)\mathrm{cos}\:\theta\right]}{\mathrm{1}+\mathrm{sin}\:\theta\mathrm{cos}\:\theta}\:−\:\left({R}−{r}\right)\right]}\:\:. \\ $$

Commented by mrW2 last updated on 03/Feb/18

The forces and also their directions  as displayed in the diagram are   correct. I can follow your steps and  they are correct.

$${The}\:{forces}\:{and}\:{also}\:{their}\:{directions} \\ $$$${as}\:{displayed}\:{in}\:{the}\:{diagram}\:{are}\: \\ $$$${correct}.\:{I}\:{can}\:{follow}\:{your}\:{steps}\:{and} \\ $$$${they}\:{are}\:{correct}. \\ $$

Commented by ajfour last updated on 04/Feb/18

thank  you Sir.

$${thank}\:\:{you}\:{Sir}. \\ $$

Terms of Service

Privacy Policy

Contact: info@tinkutara.com